Trigo dilemma

Geometry Level 3

( sec 2 1 0 + tan 1 0 ) ( sec 2 5 0 tan 5 0 ) ( sec 2 7 0 + tan 7 0 ) \left(\sec^{2}10^{\circ}+\tan 10^{\circ}\right)\left(\sec^{2} 50^{\circ} - \tan 50^{\circ}\right)\left(\sec^{2} 70^{\circ}+\tan 70^{\circ}\right)

The value of the expression above can be represented in the form a b b \dfrac{a-\sqrt{b}}{b} where a a and b b are positive coprime integers. Find the value of a + b 2 a+b^2 .


The answer is 61.

This section requires Javascript.
You are seeing this because something didn't load right. We suggest you, (a) try refreshing the page, (b) enabling javascript if it is disabled on your browser and, finally, (c) loading the non-javascript version of this page . We're sorry about the hassle.

5 solutions

Pi Han Goh
Jul 20, 2015

Apply csc ( A ) = sec ( 9 0 A ) \csc(A) = \sec(90^\circ - A) and tan ( A ) = cot ( 9 0 A ) \tan(A) = \cot(90^\circ - A) followed by cot ( A ) = cot ( 18 0 A ) \cot(A) = -\cot(180^\circ - A) , and csc 2 ( A ) = cot 2 ( A ) + 1 \csc^2(A) = \cot^2(A) + 1 .

( sec 2 1 0 + tan 1 0 ) ( sec 2 5 0 tan 5 0 ) ( sec 2 7 0 + tan 7 0 ) = ( csc 2 8 0 + cot 8 0 ) ( csc 2 4 0 cot 4 0 ) ( csc 2 2 0 + cot 2 0 ) = ( csc 2 8 0 + cot 8 0 ) ( csc 2 14 0 + cot 14 0 ) ( csc 2 2 0 + cot 2 0 ) = ( cot 2 8 0 + cot 8 0 + 1 ) ( cot 2 14 0 + cot 14 0 + 1 ) ( cot 2 2 0 + cot 2 0 + 1 ) \begin{aligned} &&\left(\sec^{2}10^{\circ}+\tan 10^{\circ}\right)\left(\sec^{2} 50^{\circ} - \tan 50^{\circ}\right)\left(\sec^{2} 70^{\circ}+\tan 70^{\circ}\right) \\ &=& \left(\csc^{2}80^{\circ}+\cot 80^{\circ}\right)\left(\csc^{2} 40^{\circ} - \cot 40^{\circ}\right)\left(\csc^{2} 20^{\circ}+\cot 20^{\circ}\right) \\ &=& \left(\csc^{2}80^{\circ}+\cot 80^{\circ}\right)\left(\csc^{2} 140^{\circ} + \cot 140^{\circ}\right)\left(\csc^{2} 20^{\circ}+\cot 20^{\circ}\right) \\ &=& \left(\cot^{2}80^{\circ}+\cot 80^{\circ}+1\right)\left(\cot^{2} 140^{\circ} + \cot 140^{\circ}+1 \right)\left(\cot^{2} 20^{\circ}+\cot 20^{\circ} + 1\right) \\ \end{aligned}

Note that

cot 2 ( A ) + cot ( A ) + 1 = sin 2 ( A ) sin 2 ( A ) ( ( cos 2 A sin 2 A ) + cos A sin A + 1 ) = 1 sin 2 ( A ) ( cos 2 ( A ) + sin ( A ) cos ( A ) + sin 2 ( A ) ) = 1 sin 2 ( A ) ( 1 + 1 2 sin ( 2 A ) ) = 1 2 sin 2 ( A ) ( sin ( 2 A ) + 2 ) \begin{aligned} \cot^2(A) + \cot(A) + 1 & =& \frac{\sin^2(A)}{\sin^2(A)} \left ( \left( \frac{\cos^2 A}{\sin^2 A} \right ) + \frac{\cos A}{\sin A} + 1 \right) \\ &=& \frac1{\sin^2(A)} ( \cos^2(A) + \sin(A) \cos(A) + \sin^2 (A) ) \\ &=& \frac1{\sin^2(A)} \left( 1 +\frac12 \sin(2A) \right ) \\ &=& \frac1{2\sin^2(A)} \left( \sin(2A) + 2\right ) \\ \end{aligned}

Then,

( sec 2 1 0 + tan 1 0 ) ( sec 2 5 0 tan 5 0 ) ( sec 2 7 0 + tan 7 0 ) = ( cot 2 8 0 + cot 8 0 + 1 ) ( cot 2 14 0 + cot 14 0 + 1 ) ( cot 2 4 0 + tan 4 0 + 1 ) = 1 8 [ ( ( sin ( 28 0 ) + 2 ) ( sin ( 16 0 ) + 2 ) ( sin ( 4 0 ) + 2 ) ] ( sin ( 2 0 ) sin ( 14 0 ) sin ( 8 0 ) ) 2 = 1 8 [ ( ( sin ( 28 0 ) + 2 ) ( sin ( 16 0 ) + 2 ) ( sin ( 4 0 ) + 2 ) ] ( sin ( 2 0 ) sin ( 14 0 ) sin ( 26 0 ) ) 2 ( ) \begin{aligned} && \left(\sec^{2}10^{\circ}+\tan 10^{\circ}\right)\left(\sec^{2} 50^{\circ} - \tan 50^{\circ}\right)\left(\sec^{2} 70^{\circ}+\tan 70^{\circ}\right) \\ &=& \left(\cot^{2}80^{\circ}+\cot 80^{\circ}+1\right)\left(\cot^{2} 140^{\circ} + \cot 140^{\circ}+1 \right)\left(\cot^{2} 40^{\circ}+\tan 40^{\circ} + 1\right) \\ &=& \frac18 \frac{\bigg [ ((\sin(280^\circ) + 2)(\sin(160^\circ) + 2)(\sin(40^\circ) + 2) \bigg ] }{(\sin(20^\circ) \sin(140^\circ) \sin(80^\circ) )^2}\\ &=& \frac18 \frac{\bigg [ ((\sin(280^\circ) + 2)(\sin(160^\circ) + 2)(\sin(40^\circ) + 2) \bigg ] }{(\sin(20^\circ) \sin(140^\circ) \sin(260^\circ) )^2} \qquad (\ast) \\ \end{aligned}

It's easier to evaluate the denominator of \ast first. We see that 20 × 3 = 60 , 140 × 3 = 360 + 60 , 260 × 3 = 2 × 360 + 60 20 \times 3 = 60, 140 \times 3 = 360 + 60, 260 \times 3 = 2\times 360 + 60 . Consider sin ( 3 x ) = 3 2 \sin(3x) = \frac{\sqrt 3}2 then angles 2 0 , 14 0 , 26 0 20^\circ, 140^\circ,260^\circ satisfy the equation. By Triple Angle formula: 4 sin 3 ( x ) + 3 sin ( x ) 3 2 = 0 -4\sin^3(x) + 3\sin(x) - \frac{\sqrt 3}2 = 0 , and by Vieta's formula, sin ( 2 0 ) sin ( 14 0 ) sin ( 26 0 ) = 3 8 \sin(20^\circ) \sin(140^\circ) \sin(260^\circ) = \frac{\sqrt 3}8 .

Similarly, for the numerator, we see that 280 × 3 = 2 × 360 + 120 , 160 × 3 = 360 + 120 , 40 × 3 = 120 280\times3= 2\times360 +120, 160\times3=360+120, 40\times3 =120 . Consider sin ( 3 y ) = 3 2 \sin(3y) = \frac{\sqrt 3}2 then angles 4 0 , 16 0 , 28 0 40^\circ, 160^\circ,280^\circ satisfy the equation. By Triple Angle formula again: 4 sin 3 ( y ) + 3 sin ( y ) 3 2 = 0 -4\sin^3(y) + 3\sin(y) - \frac{\sqrt 3}2 =0 . Letting z = sin ( y ) z = \sin(y) , then 8 z 3 6 z 3 = 0 8z^3 - 6z - \sqrt3 = 0 have roots sin ( 4 0 ) , sin ( 16 0 ) , sin ( 28 0 ) \sin(40^\circ), \sin(160^\circ), \sin(280^\circ) . Equivalently, 8 ( z 2 ) 3 6 ( z 2 ) 3 = 0 8(z-2)^3 - 6(z-2) - \sqrt3 = 0 have roots sin ( 4 0 ) + 2 , sin ( 16 0 ) + 2 , sin ( 28 0 ) + 2 \sin(40^\circ) + 2, \sin(160^\circ)+2, \sin(280^\circ) +2 . Thus by Vieta's formula, we have ( ( sin ( 28 0 ) + 2 ) ( sin ( 16 0 ) + 2 ) ( sin ( 4 0 ) + 2 ) ((\sin(280^\circ) + 2)(\sin(160^\circ) + 2)(\sin(40^\circ) + 2) equals to the negative ratio of constant term to the leading coefficient of the previous equation, which is 1 8 ( 2 3 8 + 6 2 3 ) = 1 8 ( 52 3 ) \frac18(-2^3 \cdot 8 + 6\cdot 2 - \sqrt3) = \frac18 (52- \sqrt 3) .

Putting it all together gives

( sec 2 1 0 + tan 1 0 ) ( sec 2 5 0 tan 5 0 ) ( sec 2 7 0 + tan 7 0 ) = 1 8 1 8 ( 52 3 ) ÷ ( 3 2 ) 2 = 52 3 3 \begin{aligned} &&\left(\sec^{2}10^{\circ}+\tan 10^{\circ}\right)\left(\sec^{2} 50^{\circ} - \tan 50^{\circ}\right)\left(\sec^{2} 70^{\circ}+\tan 70^{\circ}\right) \\ &=& \frac18 \cdot \frac18 (52- \sqrt 3) \div \left( \frac {\sqrt 3}2 \right)^2 = \large \boxed{\frac{52-\sqrt3}3} \\ \end{aligned}

Moderator note:

Slight typo in your first block of equations. In the second line, you didn't change tan 7 0 \tan 70 ^ \circ into cot 2 0 \cot 20 ^ \circ .

Once you get it in the form of ( x i 2 + x i + 1 ) \prod ( x_i^2 + x_i + 1 ) , there is actually a better way of evaluating this via ( x i ω ) ( x i ω 2 ) = f ( ω ) f ( ω 2 ) \prod ( x_i - \omega) ( x_i - \omega^2 ) = f( \omega ) f ( \omega^2 ) with f ( x ) = ( x x i ) f(x) = \prod ( x - x_i) .

ChallengeMaster: Interesting, let me work on that and see what I can come up with!

Pi Han Goh - 5 years, 10 months ago

Please check my solution as well.. :)

Samarth Kapoor - 5 years, 10 months ago
Aakash Khandelwal
Jul 21, 2015

THESE ARE FEW HINTS TO SOLVE THIS PROBLEM IN LESS STEPS First convert all terms into sin and cos and solve numerator and denominator separately .

For numerator: convert all trig ratios in acute angle of sine by using quadrants and identity 2sinxcosx=sin(2x). For simple calculations multiply 2 factors having positive signs and apply C and D formulaes. After simplifying multiply 3rd term . You will get numerical value except -sin20 sin40 sin80. You can solve it seperately my using sinx sin(60+ x) sin(60-x)=0.25 sin(3x). For denominator : use cosx cos(60+ x) cos(60-x)=0.25 cos(3x)

Samarth Kapoor
Jul 20, 2015

I know its a bit tedious but have a look please.

Can you explain the step of "multiply the two relations to get the answer"? Why does that work?

(I understand what you're doing, but for the sake of those of use who are not mind readers, you should explain each step of your proof clearly. That is arguably the most crucial and vital part of your proof.)

Calvin Lin Staff - 5 years, 10 months ago

Log in to reply

I may not be able to operate computer for a few days.. and I am quite busy to write up a solution right now.. So....

Samarth Kapoor - 5 years, 10 months ago

Log in to reply

No worries take your time. I'm offering suggestions for how you can improve the quality of your solution. What you have here is a good solution idea, but it has not been expressed in a way that others can easily understand it.

Calvin Lin Staff - 5 years, 10 months ago
Mas Mus
Feb 5, 2017

( sec 2 1 0 + tan 1 0 ) ( sec 2 5 0 tan 5 0 ) ( sec 2 7 0 + tan 7 0 ) \left(\sec^{2}10^{\circ}+\tan 10^{\circ}\right)\left(\sec^{2} 50^{\circ} - \tan 50^{\circ}\right)\left(\sec^{2} 70^{\circ}+\tan 70^{\circ}\right) = ( 1 + sin 1 0 cos 1 0 cos 2 1 0 ) ( 1 sin 5 0 cos 5 0 cos 2 5 0 ) ( 1 + sin 7 0 cos 7 0 cos 2 7 0 ) = ( 2 + sin 2 0 2 cos 2 1 0 ) ( 2 sin 10 0 2 cos 2 5 0 ) ( 2 + sin 14 0 2 cos 2 7 0 ) = x y \begin{aligned}\\&=\left(\frac{1+\sin10^{\circ}\cos10^{\circ}}{\cos^{2} 10^{\circ}}\right)\left(\frac{1-\sin50^{\circ}\cos50^{\circ}}{\cos^{2} 50^{\circ}}\right)\left(\frac{1+\sin70^{\circ}\cos70^{\circ}}{\cos^{2}70^{\circ}}\right)\\&=\left(\frac{2+\sin20^{\circ}}{2\cos^{2} 10^{\circ}}\right)\left(\frac{2-\sin100^{\circ}}{2\cos^{2}50^{\circ}}\right)\left(\frac{2+\sin140^{\circ}}{2\cos^{2}70^{\circ}}\right)=\frac{x}{y}\end{aligned} \\\\

Now, we evaluate x x \\\\

x = ( 2 + sin 2 0 ) ( 2 sin 10 0 ) ( 2 + sin 14 0 ) = ( 4 2 ( sin 10 0 sin 2 0 ) sin 10 0 sin 2 0 ) ( 2 + sin 14 0 ) = ( 4 4 cos 6 0 sin 4 0 + 1 2 ( cos 12 0 cos 8 0 ) ) ( 2 + sin 14 0 ) = ( 15 4 2 sin 4 0 1 2 cos 8 0 ) ( 2 + sin 14 0 ) = 15 2 + 15 4 sin 14 0 4 sin 4 0 2 sin 14 0 sin 4 0 cos 8 0 1 2 sin 14 0 cos 8 0 = 15 2 1 4 sin 4 0 2 sin 2 4 0 ( 1 2 sin 2 4 0 ) 1 4 ( sin 22 0 + sin 6 0 ) = 13 2 1 4 sin 4 0 + 1 4 sin 4 0 3 8 = 52 3 8 \begin{aligned}x&=\left(2+\sin20^{\circ}\right)\left(2-\sin100^{\circ}\right)\left(2+\sin140^{\circ}\right)\\&=\left(4-2\left(\sin100^{\circ}-\sin20^{\circ}\right)-\sin100^{\circ}\sin20^{\circ}\right)\left(2+\sin140^{\circ}\right)\\&=\left(4-4\cos60^{\circ}\sin40^{\circ}+\frac{1}{2}\left(\cos120^{\circ}-\cos80^{\circ}\right)\right)\left(2+\sin140^{\circ}\right)\\&=\left(\frac{15}{4}-2\sin40^{\circ}-\frac{1}{2}\cos80^{\circ}\right)\left(2+\sin140^{\circ}\right)\\&=\frac{15}{2}+\frac{15}{4}\sin140^{\circ}-4\sin40^{\circ}-2\sin140^{\circ}\sin40^{\circ}-\cos80^{\circ}-\frac{1}{2}\sin140^{\circ}\cos80^{\circ}\\&=\frac{15}{2}-\frac{1}{4}\sin40^{\circ}-2\sin^{2}40^{\circ}-\left(1-2\sin^{2}40^{\circ}\right)-\frac{1}{4}\left(\sin220^{\circ}+\sin60^{\circ}\right)\\&=\frac{13}{2}-\frac{1}{4}\sin40^{\circ}+\frac{1}{4}\sin40^{\circ}-\frac{\sqrt{3}}{8}=\frac{52-\sqrt{3}}{8}\end{aligned}

Now, we evaluate y y

2 cos 2 1 0 × 2 cos 2 5 0 × 2 cos 2 7 0 = 8 ( cos 1 0 × cos 5 0 × cos 7 0 ) 2 = 8 ( ( cos 6 0 + cos 4 0 ) 2 × cos 7 0 ) 2 = 8 ( 1 4 cos 7 0 + 1 4 ( cos 11 0 + cos 3 0 ) ) 2 = 8 ( 3 8 + 1 4 ( cos 11 0 + cos 7 0 ) ) 2 = 8 × 3 64 = 3 8 \begin{aligned}2\cos^{2}10^{\circ}\times2\cos^{2}50^{\circ}\times2\cos^{2}70^{\circ}&=8\left(\cos10^{\circ}\times\cos50^{\circ}\times\cos70^{\circ}\right)^{2}\\&=8\left(\frac{(\cos60^{\circ}+\cos40^{\circ})}{2}\times\cos70^{\circ}\right)^{2}\\&=8\left(\frac{1}{4}\cos70^{\circ}+\frac{1}{4}(\cos110^{\circ}+\cos30^{\circ})\right)^{2}\\&=8\left(\frac{\sqrt{3}}{8}+\frac{1}{4}(\cos110^{\circ}+\cos70^{\circ})\right)^{2}=8\times\frac{3}{64}=\frac{3}{8}\end{aligned}

Now, we have

x y = 52 3 3 = a b b \large{\frac{x}{y}=\frac{52-\sqrt{3}}{3}=\frac{a-\sqrt{b}}{b}}

Finally, a + b 2 = 61 \boxed{a+b^2=61}

Pierre Lapôtre
Sep 4, 2016

L a T e X LaTeX \begin{align } &\left(\sec^2 10°+\tan 10°\right)\left(\sec^2 50°-\tan 50°\right)\left(\sec^2 70°+\tan 70°\right)\ =&\left(\sec^2 10°+\tan 10°\right)\left(\sec^2 130°+\tan 130°\right)\left(\sec^2 70°+\tan 70°\right) \end{align }

For all $\theta \neq 90 + k\times 180, k\in Z$, $\sec^2\theta = 1 + \tan^2\theta$ so,\ \begin{align } =&\left(1+\tan 10°+\tan^2 10°\right)\left(1+\tan 130°+\tan ^2130°\right)\left(1+\tan 70°+\tan ^270°\right) \end{align } we note $\tan 10° = a, \tan 130° = b, \tan 70° = c$\

we have to calculate $(1+a+a^2)(1+b+b^2)(1+c+c^2)$\ we develop (tedious calculation) : \ $1 + (a+b+c)+(ab+ac+bc)+(ab^2+ac^2+a^2b+a^2c+b^2c+bc^2)+(a^2+b^2+c^2)\+(a^2b^2+a^2c^2+b^2c^2)+(abc)+(ab^2c+a^2bc+abc^2)+(a^2b^2c+a^2bc^2+ab^2c^2)+(a^2b^2c^2)$ (1)\ with Samarth Kapoor's idea : $(x-a)(x-b)(x-c) = x^3-\sqrt{3}x^2-3x+\dfrac{1}{\sqrt{3}}$\ we get \begin{align } a+b+c&=\sqrt{3}\ ab+ac+bc&=-3\ abc&=-\dfrac{1}{\sqrt{3}} \end{align } so we find \begin{align } a^2+b^2=c^2&=9\ ab^2+ac^2+a^2b+a^2c+bc^2+b^2c&=-2\sqrt{3}\ a^2b^2+a^2c^2+b^2c^2&=11\ ab^2c+a^2bc+abc^2&=-1\ a^2b^2c+a^2bc^2+ab^2c^2&=\sqrt{3}\ a^2b^2c^2&=\dfrac{1}{3} \end{align } Thus, we report in (1) and we get $(1+a+a^2)(1+b+b^2)(1+c+c^2)=\dfrac{52-\sqrt{3}}{3}$

0 pending reports

×

Problem Loading...

Note Loading...

Set Loading...